Iit Quiz

Download as pdf or txt
Download as pdf or txt
You are on page 1of 2

DEPARTMENT OF MATHEMATICS, IIT GUWAHATI

MA101: Mathematics I Quiz I (Maximum Marks: 10)


Date: August 26, 2011 Time: 8 am - 8:50 am
Model Solutions
1. Let A be an n n matrix such that the system of equations Ax = 0 has a non-trivial solution. Is it possible that
the system of equations A
t
x = b has a unique solution for some b R
n
? Justify. 2
Solution: Since Ax = 0 has a non-trivial solution, we have rank(A) < n. We know that rank(A
t
) = rank(A),
and so rank(A
t
) < n. If rank[A
t
| b] = rank(A
t
), then the system A
t
x = b has no solution. If rank[A
t
| b] =
rank(A
t
) < n, then the system A
t
x = b will have innitely many solutions. In either case, A
t
x = b cannot have
a unique solution.
Aliter
Since the system of equations Ax = 0 has a non-trivial solution, we nd that the matrix A is not invertible. We
also know that A is invertible i A
t
is invertible. Thus the matrix A
t
is not invertible, and hence the system of
equations A
t
x = b cannot have a unique solution for any b R
n
.
2. Examine whether the following sets are subspaces of R
2
: 1+1
(a) {[x, y]
t
: x R, y Z} (b) {[x, y]
t
R
2
: 2x +y = 0 or x + 2y = 0}.
Solution: (a) Let S = {[x, y]
t
: x R, y Z}. We see that [0, 1]
t
S and
1
2
R, but
1
2
[0, 1]
t
=

0,
1
2

t
/ S.
Hence S is not a subspace of R
2
.
(b) Let T = {[x, y]
t
R
2
: 2x + y = 0 or x + 2y = 0}. We see that [1, 2]
t
T and [2, 1]
t
T, but
[1, 2]
t
+ [2, 1]
t
= [1, 1]
t
/ T. Hence T is not a subspace of R
2
.
3. Prove or disprove: If S = {v
1
, v
2
, v
3
, v
4
, v
5
, v
6
} is a linearly dependent set of vectors in R
5
then each vector of S
must be a linear combination of the remaining vectors of S. 2
Solution: Consider the set S = {v
1
, v
2
, v
3
, v
4
, v
5
, v
6
}, where v
1
= [1, 0, 0, 0, 0]
t
, v
2
= [0, 1, 0, 0, 0]
t
, v
3
=
[0, 0, 1, 0, 0]
t
, v
4
= [0, 0, 0, 1, 0]
t
, v
5
= [0, 0, 0, 0, 1]
t
and v
6
= 0 = [0, 0, 0, 0, 0]
t
. Clearly S is a linearly dependent
set, since 0 S. However, the vectors v
1
, v
2
, v
3
, v
4
and v
5
are linearly independent and v
6
= 0. Hence v
1
is not
a linear combination of the vectors v
2
, v
3
, v
4
, v
5
and v
6
.
Thus, the given statement is disproved by the above counterexample.
4. Let A and B be two matrices of sizes mn and n m respectively and let m > n. Prove that the matrix AB is
not invertible. 2
Solution: We have rank(AB) rank(B) n < m. Since AB is an m m matrix and rank(AB) < m, we nd
that the matrix AB is not invertible.
Aliter
Since rank(B) n < m, we have that there is non-zero solution of the system Bx = 0. Let u R
m
be such that
u = 0 and Bu = 0. Then we have (AB)u = A(Bu) = A0 = 0. Thus u is also a non-zero solution of the system
(AB)x = 0. Hence the matrix AB is not invertible.
5. Find a basis for the subspace spanned by the vectors [2, 1, 3, 1]
t
, [1, 2, 0, 1]
t
, [0, 2, 2, 1]
t
and [1, 1, 1, 1]
t
. 2
Solution: We have

1 2 0 1
2 1 3 1
0 2 2 1
1 1 1 1

R
2
R
2
2R
1

1 2 0 1
0 3 3 1
0 2 2 1
1 1 1 1

R
4
R
4
R
1

1 2 0 1
0 3 3 1
0 2 2 1
0 1 1 0

R
2

1
3
R
2

1 2 0 1
0 1 1
1
3
0 2 2 1
0 1 1 0

R
3
R
3
+ 2R
2

1 2 0 1
0 1 1
1
3
0 0 0
1
3
0 1 1 0

R
4
R
4
+R
2

1 2 0 1
0 1 1
1
3
0 0 0
1
3
0 0 0
1
3

R
4
R
4
+R
3

1 2 0 1
0 1 1
1
3
0 0 0
1
3
0 0 0 0

The last matrix is in row-echelon form. Hence a basis for the subspace spanned by the given vectors is

[1, 2, 0, 1]
t
, [0, 1, 1,
1
3
]
t
, [0, 0, 0,
1
3
]
t

.
End

You might also like

pFad - Phonifier reborn

Pfad - The Proxy pFad of © 2024 Garber Painting. All rights reserved.

Note: This service is not intended for secure transactions such as banking, social media, email, or purchasing. Use at your own risk. We assume no liability whatsoever for broken pages.


Alternative Proxies:

Alternative Proxy

pFad Proxy

pFad v3 Proxy

pFad v4 Proxy